2025 AMC 12A Problems/Problem 25: Difference between revisions
Tiankaizhang (talk | contribs) |
Tiankaizhang (talk | contribs) Undo revision 266770 by Tiankaizhang (talk) Tag: Undo |
||
| Line 1: | Line 1: | ||
Polynomials <imath>P(x)</imath> and <imath>Q(x)</imath> each have degree <imath>3</imath> and leading coefficient <imath>1</imath>, and their roots are all elements of <imath>\{1,2,3,4,5\}</imath>. The function <imath>f(x) = \tfrac{P(x)}{Q(x)}</imath> has the property that there exist real numbers <imath>a < b < c < d</imath> such that the set of all real numbers <imath>x</imath> such that <imath>f(x) \leq 0</imath> consists of the closed interval <imath>[a,b]</imath> together with the open interval <imath>(c,d)</imath>. How many functions <imath>f(x)</imath> are possible? | Polynomials <imath>P(x)</imath> and <imath>Q(x)</imath> each have degree <imath>3</imath> and leading coefficient <imath>1</imath>, and their roots are all elements of <imath>\{1,2,3,4,5\}</imath>. The function <imath>f(x) = \tfrac{P(x)}{Q(x)}</imath> has the property that there exist real numbers <imath>a < b < c < d</imath> such that the set of all real numbers <imath>x</imath> such that <imath>f(x) \leq 0</imath> consists of the closed interval <imath>[a,b]</imath> together with the open interval <imath>(c,d)</imath>. How many functions <imath>f(x)</imath> are possible? | ||
We | ==Solution 1 == | ||
We begin by analyzing the sign chart. From the condition <imath>f(x) \le 0</imath> on <imath>[a,b] \cup (c,d)</imath> and <imath>f(x) > 0</imath> elsewhere, we deduce the following: | |||
- <imath>a</imath> and <imath>b</imath> are zeros of <imath>f</imath>, since we transition from <imath>f>0</imath> to <imath>f\le0</imath> at <imath>a</imath>, and from <imath>f\le0</imath> to <imath>f>0</imath> at <imath>b</imath>. | |||
- <imath>c</imath> and <imath>d</imath> are poles or holes of <imath>f</imath>, since on <imath>(c,d)</imath> we have <imath>f\le0</imath>, while immediately outside that interval <imath>f>0</imath>. | |||
Thus, the sign pattern of <imath>f(x)</imath> is | |||
<cmath> | <cmath> | ||
\{ | \begin{array}{cccccccccc} | ||
& (-\infty, a) & a & (a,b) & b & (b,c) & c & (c,d) & d & (d,\infty) \\ | |||
f(x) & + & 0 & - & 0 & + & \text{pole} & - & \text{pole} & + | |||
\end{array} | |||
</cmath> | </cmath> | ||
--- | --- | ||
**Step 2. Structure of <imath>f(x)</imath>** | |||
According to the given conditions, we can express | |||
<cmath> | |||
f(x) = \frac{(x-p_1)(x-p_2)(x-p_3)}{(x-q_1)(x-q_2)(x-q_3)}. | |||
</cmath> | |||
Combining this with the sign analysis, we can write | |||
<cmath> | |||
f(x) = \frac{(x-a)(x-b)(x-p_3)}{(x-c)(x-d)(x-q_3)}. | |||
</cmath> | |||
We can separate this into two factors: | |||
<cmath> | |||
f(x) = \frac{(x-a)(x-b)}{(x-c)(x-d)} \cdot \frac{x-p_3}{x-q_3}. | |||
</cmath> | |||
Define | |||
<cmath> | <cmath> | ||
( | f_1(x) = \frac{(x-a)(x-b)}{(x-c)(x-d)}, \qquad f_2(x) = \frac{x-p_3}{x-q_3}. | ||
</cmath> | </cmath> | ||
--- | --- | ||
**Step 3. Determining <imath>p_3</imath> and <imath>q_3</imath>** | |||
By analyzing the sign changes of <imath>f_1(x)</imath>, we see that it already matches the required sign pattern for <imath>f(x)</imath>. Therefore, <imath>f_2(x)</imath> must remain positive on every interval; otherwise, it would introduce extra sign changes. | |||
This means <imath>p_3 = q_3</imath>, since if <imath>p_3 \ne q_3</imath>, the factor <imath>\frac{x-p_3}{x-q_3}</imath> would change sign between <imath>p_3</imath> and <imath>q_3</imath>. | |||
Let <imath>p_3 = q_3 = t</imath>. | |||
Furthermore: | |||
- <imath>t</imath> cannot lie within <imath>[a,b] \cup (c,d)</imath>, since that would alter the set <imath>\{x : f(x) \le 0\}</imath>. | |||
- <imath>t</imath> cannot be equal to <imath>a</imath> or <imath>b</imath>, since that would make those endpoints holes (discontinuities), contradicting that <imath>[a,b]</imath> is a *closed* interval. | |||
Hence, <imath>t</imath> must either be equal to <imath>c</imath> or <imath>d</imath>, or lie outside <imath>[a,b] \cup (c,d)</imath>. | |||
--- | |||
**Step 4. Possible values of <imath>a,b,c,d,t</imath>** | |||
Given <imath>a<b<c<d</imath> and <imath>\{x : f(x) \le 0\} = [a,b] \cup (c,d)</imath>, we choose four distinct numbers from <imath>\{1,2,3,4,5\}</imath> for <imath>a,b,c,d</imath>: | |||
<cmath> | |||
\binom{5}{4} = 5. | |||
</cmath> | |||
The five possible cases are: | |||
<cmath> | <cmath> | ||
[1,2]\cup(3,4), \quad [1,2]\cup(3,5), \quad [1,2]\cup(4,5), \quad [1,3]\cup(4,5), \quad [2,3]\cup(4,5). | |||
</cmath> | </cmath> | ||
--- | --- | ||
**Case 1:** <imath>[1,2]\cup(3,4)</imath> | |||
<imath>t</imath> can be <imath>3</imath>, <imath>4</imath>, or <imath>5</imath>, giving 3 functions: | |||
<cmath> | <cmath> | ||
\ | f(x) = \frac{(x-1)(x-2)(x-3)}{(x-3)(x-4)(x-3)}, \quad | ||
f(x) = \frac{(x-1)(x-2)(x-4)}{(x-3)(x-4)(x-4)}, \quad | |||
f(x) = \frac{(x-1)(x-2)(x-5)}{(x-3)(x-4)(x-5)}. | |||
</cmath> | </cmath> | ||
giving | |||
**Case 2:** <imath>[1,2]\cup(3,5)</imath> | |||
<imath>t</imath> can be <imath>3</imath> or <imath>5</imath>, giving 2 functions: | |||
<cmath> | <cmath> | ||
f(x) = \frac{(x-1)(x-2)(x-3)}{(x-3)(x-5)(x-3)}, \quad | |||
f(x) = \frac{(x-1)(x-2)(x-5)}{(x-3)(x-5)(x-5)}. | |||
</cmath> | </cmath> | ||
**Case 3:** <imath>[1,2]\cup(4,5)</imath> | |||
<imath>t</imath> can be <imath>3</imath>, <imath>4</imath>, or <imath>5</imath>, giving 3 functions: | |||
<cmath> | |||
f(x) = \frac{(x-1)(x-2)(x-3)}{(x-4)(x-5)(x-3)}, \quad | |||
f(x) = \frac{(x-1)(x-2)(x-4)}{(x-4)(x-5)(x-4)}, \quad | |||
f(x) = \frac{(x-1)(x-2)(x-5)}{(x-4)(x-5)(x-5)}. | |||
</cmath> | |||
**Case 4:** <imath>[1,3]\cup(4,5)</imath> | |||
<imath>t</imath> can be <imath>4</imath> or <imath>5</imath>, giving 2 functions: | |||
<cmath> | |||
f(x) = \frac{(x-1)(x-3)(x-4)}{(x-4)(x-5)(x-4)}, \quad | |||
f(x) = \frac{(x-1)(x-3)(x-5)}{(x-4)(x-5)(x-5)}. | |||
</cmath> | |||
**Case 5:** <imath>[2,3]\cup(4,5)</imath> | |||
<imath>t</imath> can be <imath>1</imath>, <imath>4</imath>, or <imath>5</imath>, giving 3 functions: | |||
<cmath> | <cmath> | ||
3 | f(x) = \frac{(x-2)(x-3)(x-1)}{(x-4)(x-5)(x-1)}, \quad | ||
f(x) = \frac{(x-2)(x-3)(x-4)}{(x-4)(x-5)(x-4)}, \quad | |||
f(x) = \frac{(x-2)(x-3)(x-5)}{(x-4)(x-5)(x-5)}. | |||
</cmath> | </cmath> | ||
--- | --- | ||
<imath>\boxed{13}</imath> possible functions <imath>f(x)</imath>, corresponding to | **Step 5. Final Answer** | ||
Summing all possible cases: | |||
<cmath> | |||
3 + 2 + 3 + 2 + 3 = 13. | |||
</cmath> | |||
Thus, there are <imath>\boxed{13}</imath> possible functions <imath>f(x)</imath>, corresponding to choice <imath>\boxed{E}</imath>. | |||
-Victor Zhang | |||
==Video Solution 1 by OmegaLearn== | ==Video Solution 1 by OmegaLearn== | ||
https://youtu.be/SPbTyq3Dz_0 | https://youtu.be/SPbTyq3Dz_0 | ||
Revision as of 02:27, 7 November 2025
Polynomials
and
each have degree
and leading coefficient
, and their roots are all elements of
. The function
has the property that there exist real numbers
such that the set of all real numbers
such that
consists of the closed interval
together with the open interval
. How many functions
are possible?
Solution 1
We begin by analyzing the sign chart. From the condition
on
and
elsewhere, we deduce the following:
-
and
are zeros of
, since we transition from
to
at
, and from
to
at
.
-
and
are poles or holes of
, since on
we have
, while immediately outside that interval
.
Thus, the sign pattern of
is
---
- Step 2. Structure of
**
- Step 2. Structure of
According to the given conditions, we can express
Combining this with the sign analysis, we can write
We can separate this into two factors:
Define
---
- Step 3. Determining
and
**
- Step 3. Determining
By analyzing the sign changes of
, we see that it already matches the required sign pattern for
. Therefore,
must remain positive on every interval; otherwise, it would introduce extra sign changes.
This means
, since if
, the factor
would change sign between
and
.
Let
.
Furthermore:
-
cannot lie within
, since that would alter the set
.
-
cannot be equal to
or
, since that would make those endpoints holes (discontinuities), contradicting that
is a *closed* interval.
Hence,
must either be equal to
or
, or lie outside
.
---
- Step 4. Possible values of
**
- Step 4. Possible values of
Given
and
, we choose four distinct numbers from
for
:
The five possible cases are:
---
- Case 1:**
![$[1,2]\cup(3,4)$](//latex-new.aopstest.com/c/d/1/cd1fd85db1157a27ef3aa4184d8d421ad6206778.png)
- Case 1:**
can be
,
, or
, giving 3 functions:
- Case 2:**
![$[1,2]\cup(3,5)$](//latex-new.aopstest.com/c/c/9/cc9e0b4dcb6f68b5ee589b76d0af05d0b8f6cedd.png)
- Case 2:**
can be
or
, giving 2 functions:
- Case 3:**
![$[1,2]\cup(4,5)$](//latex-new.aopstest.com/1/b/a/1ba35087a58df324622074c4d942fb35502adc56.png)
- Case 3:**
can be
,
, or
, giving 3 functions:
- Case 4:**
![$[1,3]\cup(4,5)$](//latex-new.aopstest.com/5/9/d/59dfdb005350637a5e72542dc1507abec272dc54.png)
- Case 4:**
can be
or
, giving 2 functions:
- Case 5:**
![$[2,3]\cup(4,5)$](//latex-new.aopstest.com/5/9/a/59adf58ab80ba15a2e8d9d51aa3ec5cee5ea29a3.png)
- Case 5:**
can be
,
, or
, giving 3 functions:
---
- Step 5. Final Answer**
Summing all possible cases:
Thus, there are
possible functions
, corresponding to choice
.
-Victor Zhang